LSAT and Law School Admissions Forum

Get expert LSAT preparation and law school admissions advice from PowerScore Test Preparation.

 Administrator
PowerScore Staff
  • PowerScore Staff
  • Posts: 8916
  • Joined: Feb 02, 2011
|
#23470
Complete Question Explanation

Flaw in the Reasoning-#%. The correct answer choice is (B)

In this stimulus the author alleges political preference on the part of reporters, who voted overwhelmingly (89% of them) for the incumbent in the last election. The author asserts that this bias manifest itself in news program content, which was negative on the challenger about 54% of the time, and negative on the incumbent only 30% of the time.

What is the problem with this argument? If these two acted exactly the same, then the news programs would clearly be biased. If, however, most of the news about the challenger was empirically negative (as in the challenger was involved in more scandals, for example), then this would explain why we would see such a high percentage of negative stories about the challenger in the news.

Answer choice (A): The argument doesn't presume that they receive equal amounts of coverage, but rather that there are equal proportions of positive versus negative news stories between the candidates. One way to see this answer as problematic is that the stimulus uses percentage information, which automatically disconnects the analysis from actual numbers (and actual numbers are what is needed to justify the "amounts' mentioned in this answer).

Answer choice (B): This is the correct answer choice. The author doesn't consider that maybe the challenger is giving the media more negative material to work with—maybe the challenger has been involved in a number of shady business deals or had a scandalous relationship that came to light during the election. Unequal coverage can be justified if the actual actions of each candidate are demonstrably different. This might even be why the reporters voted so much for the incumbent in the last election.

Answer choice (C): The author does not make such a value judgment, does not claim that it is detrimental, but rather merely asserts (incorrectly) that bias does have an effect on negative news reporting.

Answer choice (D): The argument is not concerned with effect on the electorate, but just on whether bias can affect relatively how much negative news is reported on a candidate.

Answer choice (E): The source of the reporters' preference for the incumbent is not important—we know that they showed preference in the last election. The relevant question here is whether or not that preference caused bias in the news reporting.
 karunyavgopal
  • Posts: 3
  • Joined: Dec 29, 2016
|
#34668
Could you please explain why A is wrong? If the two candidates did not receive equal amounts of coverage overall and the incumbent received more coverage overall than the challenger did, wouldn't it be possible that the 30% of negative coverage re the incumbent corresponded to a higher amount of negative coverage than that of the challenger? Thank you!
 Francis O'Rourke
PowerScore Staff
  • PowerScore Staff
  • Posts: 471
  • Joined: Mar 10, 2017
|
#34712
It is possible that the two candidates did not receive equal amounts of coverage overall. However we do not know that for certain since the stimulus used percentages, not hard numbers.

Flaw in the Reasoning questions, like this, require you to accurately describe what the speaker said. In this way, Flaw questions are similar to Must Be True questions: if Answer Choice (A) attributes something to the speaker that cannot be proven using the stimulus, we must eliminate it.
 Sophia123
  • Posts: 43
  • Joined: Mar 20, 2017
|
#40089
Hello!

I am slightly confused about the explanations for C and D - I eliminated C because "always" just seemed too strong of a word, but is there any way the original explanation for why this answer is incorrect can be restated in a different way?

I also kept D as a contender in thinking that if the electorate doesn't care what the news coverage is, how can the bias have any impact on the outcome? Where did I go wrong in this logic?

Thank you in advance!

-Sophia
User avatar
 Dave Killoran
PowerScore Staff
  • PowerScore Staff
  • Posts: 5853
  • Joined: Mar 25, 2011
|
#40094
Hi Sophia,

Thanks for the questions! I quoted you below in order to respond directly to each part of your post :-D

Sophia123 wrote:I eliminated C because "always" just seemed too strong of a word, but is there any way the original explanation for why this answer is incorrect can be restated in a different way?
I think it's a slight stretch to say the author here thinks that allowing biases to influence reporting is detrimental, mainly because the author doesn't make a direct statement about it. So, we have a problem on that basis alone. Then as you quite rightly note, is it further presumed that this is "always" an issue? No, we don't have any information on that at all, and so this answer has problems.

Sophia123 wrote:I also kept D as a contender in thinking that if the electorate doesn't care what the news coverage is, how can the bias have any impact on the outcome? Where did I go wrong in this logic?
Here's the thing about your analysis: it doesn't really relate to what the author said. While your point makes sense (hey, if no one is watching/reading/listening, who cares what biases might exist?), take a close look at the point being made in the argument: it's simply that the biases for the reporters were then reflected in the stories they chose to present. That doesn't presume anything about how the electorate acts or whether the news reporting has any effect, making this answer incorrect.


Please let me know if this helps. Thanks!
 NolaB
  • Posts: 6
  • Joined: Jan 29, 2018
|
#45302
Francis O'Rourke wrote:It is possible that the two candidates did not receive equal amounts of coverage overall. However we do not know that for certain since the stimulus used percentages, not hard numbers.

Flaw in the Reasoning questions, like this, require you to accurately describe what the speaker said. In this way, Flaw questions are similar to Must Be True questions: if Answer Choice (A) attributes something to the speaker that cannot be proven using the stimulus, we must eliminate it.
Sorry to say I'm still struggling with the distinction between (A) and (B), and this answer didn't help me. Maybe someone can clarify?

Let's say (A) is wrong because we can't know for certain that the candidates did not receive equal amounts of coverage. Isn't (B) open to the same criticism, because we can't know for certain that there was more negative news worthy of reporting concerning the challenger? Does this have something to do with the use of "presumes" in (A) vs. "ignores the possibility" in (B)?

I know I'm probably digging too deep into this, but I often find myself struggling to make this distinction on Flaw questions. When an answer choice suggests the author "ignores the possibility" of something, I am often hesitant to accept the answer choice, for fear of it being out of scope. "Ignores the possibility" raises my "new information" red flags, which can be problematic in Must Be True question types!

Thanks so much,

Nola
 Alex Bodaken
PowerScore Staff
  • PowerScore Staff
  • Posts: 136
  • Joined: Feb 21, 2018
|
#45397
NolaB,

Okay, let's get into (A) vs. (B). (A) says the author "presumes, without providing justification, that both candidates received equal amounts of coverage overall." Do we know the author did that? We don't, because the author specifically speaks in percentages, not absolute terms. So, quite simply, this isn't a flaw the author makes.

(B) is something different, it reads that the author "ignores the possibility that there was more negative news worthy of reporting concerning the challenger than there was concerning the incumbent." Well, we can try this out - if there were more negative news worthy of reporting for the challenger than the incumbent, it would follow that the different percentages of relative negative coverage may not be driven by a bias, but simply by the events as they unfold. Therefore, it would be a flaw in the reasoning.

To quickly speak to your more general thought about "ignoring the possibility" - I think the way to approach this is to look at answer choices with this stem and think, first, "Is this something the author may have ignored?" and if that's possible, think "if this were true, would it be a flaw?" If the answer to both of those questions is yes, you should feel confident picking the answer, as it represents a flaw in the reasoning.

Hope that helps!
Alex
User avatar
 jrschultz14
  • Posts: 9
  • Joined: Aug 02, 2021
|
#89284
Administrator wrote: Wed Apr 27, 2016 9:33 am Answer choice (E): The source of the reporters' preference for the incumbent is not important—we know that they showed preference in the last election. The relevant question here is whether or not that preference caused bias in the news reporting.
Hi, quick question for AC E. In the stimulus, the conclusion of the argument states "reporters allowed the personal biases reflected in this voting pattern to effect their news coverage." This seems to suggest that the conclusion specifically is that the reporters' political interests is the reason for their biased coverage, and not anything else.

But wouldn't AC E be proposing an alternative reason for their biased coverage? That the bias comes from a fear of losing access required for their job rather than any political preferences for the candidates? Can you please explain why this is incorrect? Thank you!
 Robert Carroll
PowerScore Staff
  • PowerScore Staff
  • Posts: 1787
  • Joined: Dec 06, 2013
|
#89369
jr,

But that conclusion says "personal biases reflected in this voting pattern". If the reporters are giving more coverage to incumbents because they have more fear of losing access to them than to challengers, then there is a bias in their coverage because of a bias in their personal interests. Nothing in that conclusion said "political interest" - it said "personal biases". Those personal biases could be personal political views or anything. So the conclusion seems to cover the potential that the personal bias was professional rather than strictly political. Thus, the argument is not making any mistake mentioned by answer choice (E), because the conclusion covers the possibility that the personal bias was due to professional concerns about access.

Robert Carroll

Get the most out of your LSAT Prep Plus subscription.

Analyze and track your performance with our Testing and Analytics Package.